Class 9 Homework-

If \(G\) is a group and \(H,K\) are two subgroups of finite indices in \(G\), prove that \(H \cap K\) is of finite index in \(G\). Can you find an upper bound for the index of \(H \cap K\) in \(G\)?




Related Questions